Newspaper editor: Law enforcement experts, as well as most citizens, have finally come to recognize that legal pro...

Batman on November 4, 2014

Help

Please explain why (a) is the answer.

Reply
Create a free account to read and take part in forum discussions.

Already have an account? log in

Naz on November 11, 2014

This is a strengthen with sufficient premise question. Remember that asufficient premise is sufficient for a conclusion, if and only if the existence of the premise guarantees or brings about the existence of the conclusion. Therefore, we need to find the premise that 100% guarantees the conclusion. The way you want to attack these answer choices is two-pronged. Ask yourself, does it strengthen? If it doesn't, then cross it out and continue to the next answer choice. If it does strengthen, however, then ask yourself whether or not the premise guarantees the conclusion.

Conclusion: there should be no legal prohibition against gambling.

Why? No matter how diligent the effort, the laws are impossible to enforce.

So, if it is a law, then it is impossible to enforce.

P1: L ==> IE
not IE ==> not L

When a law fails to be effective, it should not be a law.

So, if a law fails to be effective, it should not be a law.

P2: not E ==> not BL
BL ==> E

A legal prohibition against gambling is a law. Therefore, a legal prohibition against gambling is impossible to enforce.

Answer choice (A) states: "No effective law is unenforceable."

So, if it is an effective law, then it is enforceable.

(A): E ==> not IE
IE ==> not E

So, does this answer choice strengthen? Yes. We know that a legal prohibition against gambling is a law, therefore--according to "P1"--it is impossible to enforce. We can then connect that to the contrapositive of (A) like so: prohibition against gambling ==> IE ==> not E. We can infer from this: prohibition against gambling ==> not E. We can connect this to P2 like so: prohibition against gambling ==> not E ==> not BL. From this we can conclude: prohibition against gambling ==> not BL, i.e. a prohibition against gambling should not be a law.

Thus, answer choice (A) strengthens the argument by connecting the premise of the argument to the conclusions.

Does answer choice (A) guarantee the conclusion of the argument? Yes. As you can see above, by using the transitive property, we connected legal prohibition against gambling to the contrapositive of (A) to P2 to conclude: a prohibition against gambling should not be a law, which is the conclusion: there should be no law against gambling.

Hope that clears things up! Please let us know if you have any other questions.